Prove that the logarithmic function is continuous on R.

Click For Summary
The discussion focuses on proving the continuity of the logarithmic function f(x) = log_a(x) for all x > 0. A user attempts a direct proof using the epsilon-delta definition of continuity but struggles with deriving the correct delta values. Another participant suggests rewriting the expression as log(x/x_0) to simplify the proof and emphasizes the importance of considering the bounds of the interval. The conversation highlights the need for careful handling of inequalities and the relationship between x and x_0 to establish continuity. Ultimately, the user gains clarity on the approach needed to complete the proof successfully.
Portuga
Messages
56
Reaction score
6

Homework Statement



Prove that f\left(x\right)=\log_{a}x is continuous for all \mathbb{R}.

Homework Equations

[/B]

I must find a \delta>0\in\mathbb{R} for a given \varepsilon>0
such that
<br /> \left|x-x_{0}\right|&lt;\delta\Rightarrow\left|\log_{a}x-\log_{a}x_{0}\right|&lt;\varepsilon.<br />

The Attempt at a Solution

.[/B]

I tried to use a direct proof solving \left|\log_{a}x-\log_{a}x_{0}\right|&lt;\varepsilon for x. But this gives rise to a pair of values: \delta=x_{0}\left(a^{\varepsilon}-1\right) and \delta=x_{0}\left(a^{-\varepsilon}-1\right). When I use them to build \left|x-x_{0}\right| &lt; \delta from the inequality
<br /> -\delta&lt;x-x_{0}&lt;\delta<br />
I see myself in big trouble, as there is no way to generate \left|\log_{a}x-\log_{a}x_{0}\right| &lt; \varepsilon.

Can someone give me a hint, or a new strategy? I have searched for help in other forums, but hints are very sophisticated to follow.
Thanks in advance.
 
Physics news on Phys.org
How do you expect a function that is not defined at ##x =0## to be continuous on the entire real line?
 
  • Like
Likes Portuga
Orodruin said:
How do you expect a function that is not defined at ##x =0## to be continuous on the entire real line?
Oh, sorry, I forgot it! Continuous for all x&gt;0.
 
Why don't you try and see where ##\log x - \log x_0 = \log \frac{x}{x_0}## gets you? Those proofs can often be done by writing it the wrong way and start with ##|f(x)-f(x_0)| < \varepsilon## to obtain a condition for ##\delta## to get an idea and then turn the estimations, resp. conclusions around.
 
You have solved <br /> -\epsilon &lt; \log_a (x/x_0) &lt; \epsilon correctly to get <br /> x_0(a^{-\epsilon} - 1) &lt; x - x_0 &lt; x_0(a^{\epsilon} - 1), but the interval is not symmetric about zero.

In this situation the closest end point to zero gives you your \delta. For suppose -b &lt; x - x_0 &lt; c for strictly positive b and c. If b &lt; c then |x - x_0| &lt; b implies -b &lt; x - x_0 &lt; b &lt; c as required; alternatively if b &gt; c then |x - x_0| &lt; c implies -b &lt; -c &lt; x - x_0 &lt; c, again as required.
 
Thank you very much, pasmith! I was so close to the answer! Now I understand. Thank you very much.
 
Question: A clock's minute hand has length 4 and its hour hand has length 3. What is the distance between the tips at the moment when it is increasing most rapidly?(Putnam Exam Question) Answer: Making assumption that both the hands moves at constant angular velocities, the answer is ## \sqrt{7} .## But don't you think this assumption is somewhat doubtful and wrong?

Similar threads

  • · Replies 105 ·
4
Replies
105
Views
6K
  • · Replies 6 ·
Replies
6
Views
2K
  • · Replies 5 ·
Replies
5
Views
2K
  • · Replies 12 ·
Replies
12
Views
2K
Replies
4
Views
2K
  • · Replies 58 ·
2
Replies
58
Views
5K
  • · Replies 13 ·
Replies
13
Views
2K
  • · Replies 3 ·
Replies
3
Views
6K
Replies
8
Views
1K
  • · Replies 4 ·
Replies
4
Views
2K